strengthening the argument

This topic has expert replies
Master | Next Rank: 500 Posts
Posts: 399
Joined: Wed Apr 15, 2009 3:48 am
Location: india
Thanked: 39 times

strengthening the argument

by xcusemeplz2009 » Sun Nov 08, 2009 12:48 am
If the public library shared by the adjacent towns of Redville and Glenwood were relocated from the library's current, overcrowded building in central Redville to a larger, available building in central Glenwood, the library would then be within walking distance of a larger number of library users. That is because there are many more people living in central Glenwood than in central Redville, and people generally will walk to the library only if it is located close to their homes.
10. Which one of the following, if true, most strengthens the argument?
(A) The public library was located between Glenwood and Redville before being moved to its current location in central Redville.
(B) The area covered by central Glenwood is approximately the same size as that covered by central Redville.
(C) The building that is available in Glenwood is smaller than an alternative building that is available in Redville.
(D) Many of the people who use the public library do not live in either Glenwood or Redville.
(E) The distance that people currently walk to get to the library is farther than what is generally considered walking distance
[spoiler]
i found E more appropriate than the OA however the OA luks to be confusing[/spoiler]
help to solve this
It does not matter how many times you get knocked down , but how many times you get up

User avatar
Senior | Next Rank: 100 Posts
Posts: 58
Joined: Tue Sep 30, 2008 10:19 am
Thanked: 7 times
GMAT Score:630

by raghavakumar85 » Sun Nov 08, 2009 1:12 am
Oh! Tough One! I am stuck between B and E.

B looks tricky while E looks correct to me.

User avatar
Senior | Next Rank: 100 Posts
Posts: 58
Joined: Tue Sep 30, 2008 10:19 am
Thanked: 7 times
GMAT Score:630

by raghavakumar85 » Sun Nov 08, 2009 1:41 am
Let me try again!

The stimulus says "moving the library from central redville to central glenwood would make it WALKING DISTANCE for LARGER number of users". So, it can be assumed that the current location of the library (central redville) is not walking distance for MANY but may be for SOME. Now, the last sentence that "since there are more number of people in glenwood than there are in redville, people will walk to the libraries closer to their homes"

So, the correct choice should support that there are more people in glenwood who are using the library and moving the library closer to glenwood makes more people to visit the library.

a. we need not know the previos location of library to serve our purpose. Elimitated.

b. Tricky.. From the stimulus we know that population of central glenwood is more than that of central redville. Suppose population of is C.R = 10 for area X and population of C.G =10 for same area X . then we can state that desnity of people who can access library in central glenwood is more because it becomes their walking distance. More the population density in C.G, nearer the library, more the people walking to libray. Hence supports our premise that LARGER people will use the library if moved to C.G (not just only because of more population) Definitely STRENGTHENS!

c. new libray may be smaller in CG, but definitely is accessible and will be in walking distance.Goes against the premise. Eliminated.

d. This choice goes against the assumption and premise. ELIMINATED.

e. I thought this could the answer. But, what if there is a possibility that people of glenwood may prefer walking to the library in central glenwood from their homes that are far from central glenwood. This raises a need to know population spread/ density in C.G. The fact that less number of people's homes are near CG or more number of people's homes are near CG is UNKNOWN to us to come to a conclusion that it can strengthen the stimulus.

Master | Next Rank: 500 Posts
Posts: 371
Joined: Fri Mar 06, 2009 2:48 am
Thanked: 27 times
GMAT Score:740

by 2010gmat » Sun Nov 08, 2009 5:58 am
i dont like any of the choice... :shock:

even if more people live in CG it does not show that more people in CG actually go to the library...

either this one is a very tricky ques or not a correct one

User avatar
Senior | Next Rank: 100 Posts
Posts: 58
Joined: Tue Sep 30, 2008 10:19 am
Thanked: 7 times
GMAT Score:630

by raghavakumar85 » Sun Nov 08, 2009 6:05 am
I agree with u gmat2010,

But in this question, i felt nothing wrong in assuming that more the people in CG, more are the people visiting library. I felt so because this assumption is not effecting any other choices. If I have to select 2 answers I would go with B and E. Because it is only 1, I did opt for B.

Master | Next Rank: 500 Posts
Posts: 371
Joined: Fri Mar 06, 2009 2:48 am
Thanked: 27 times
GMAT Score:740

by 2010gmat » Sun Nov 08, 2009 6:16 am
i agree with you that the best choice here is B...but somehow i felt bedazzled after reading the choices...
:twisted:

wats the source of this ques??

Legendary Member
Posts: 2326
Joined: Mon Jul 28, 2008 3:54 am
Thanked: 173 times
Followed by:2 members
GMAT Score:710

by gmatmachoman » Sun Nov 08, 2009 6:20 am
IMO B.

Yeah...Its extremely tricky....I do go with the concept of Population density.

Legendary Member
Posts: 1161
Joined: Mon May 12, 2008 2:52 am
Location: Sydney
Thanked: 23 times
Followed by:1 members

by mehravikas » Sun Nov 08, 2009 5:20 pm
E should be the correct answer. As it is mentioned in the argument that if the library is shifted to CG, then the library would then be within walking distance of a larger number of library users.

E states that currently the distance covered by people is farther than what is generally considered as a walking distance.

Master | Next Rank: 500 Posts
Posts: 256
Joined: Mon Aug 10, 2009 6:31 pm
Thanked: 3 times

by gmatv09 » Sun Nov 08, 2009 6:25 pm
Initially thought E was the correct choice ... :)

However, after read the last line "people generally will walk to the library only if it is located close to their homes."
I am more inclined towards choice B :roll:

Master | Next Rank: 500 Posts
Posts: 399
Joined: Wed Apr 15, 2009 3:48 am
Location: india
Thanked: 39 times

by xcusemeplz2009 » Tue Nov 10, 2009 5:42 am
OA B

source is obvious LSAT- i should rather say a demotivating source.
It does not matter how many times you get knocked down , but how many times you get up

Master | Next Rank: 500 Posts
Posts: 399
Joined: Wed Apr 15, 2009 3:48 am
Location: india
Thanked: 39 times

by xcusemeplz2009 » Tue Nov 10, 2009 5:46 am
OA B

source is obvious LSAT- i should rather say a demotivating source.
It does not matter how many times you get knocked down , but how many times you get up

Master | Next Rank: 500 Posts
Posts: 199
Joined: Sat Oct 24, 2009 4:43 pm
Thanked: 22 times
GMAT Score:710

by palvarez » Tue Nov 10, 2009 9:12 am
E is another nice trap. See what it does: it changes the referent of "walking distance" across time.

Master | Next Rank: 500 Posts
Posts: 200
Joined: Sat Aug 22, 2015 10:27 am

by james33 » Sun May 15, 2016 10:16 pm
I would go for E as well. It seems the best and safest among the rest.